Evolve HESI Leadership/Management

Lakukan tugas rumah & ujian kamu dengan baik sekarang menggunakan Quizwiz!

The night nurse receives a call at 4 AM from the laboratory regarding a client's blood cultures that have tested positive for bacteria. Which action by the nurse is appropriate at this time? 1. Call the answering service and speak to the health care provider now 2. Document the results of the culture in the client's medical record 3. Leave a message on the health care provider's office phone 4. Speak to the health care provider on rounds in the morning

1 Critical laboratory results (eg, positive blood cultures, severe electrolyte derangements) require immediate intervention for client safety. The nurse receiving a critical laboratory result should notify the health care provider (HCP) as soon as possible. Hospital organizations have individual policies regarding the time frame for notification of the HCP and HCP response, usually ≤60 minutes. Bacteremia requires timely treatment to prevent further complications (eg, septic shock) (Option 1). (Option 2) The critical laboratory result should be documented in the client's medical record, but only after immediate communication with the HCP. (Option 3) The nurse must make direct contact, either via telephone or in person, when reporting a critical result. A telephone message may not be received promptly, and a critical value requires immediate intervention. (Option 4) Even if the HCP usually makes rounds early in the morning, a critical value requires immediate, real-time notification to prevent delay of potentially urgent intervention. Educational objective: Critical laboratory results, such as positive blood cultures, require immediate communication with the health care provider (HCP) and timely intervention for client safety. The nurse must contact the HCP directly as soon as possible to avoid life-threatening complications (eg, septic shock).

A registered nurse (RN) is sent to a rehabilitation center because of chemical impairment issues. Which member of the healthcare team will be asked to provide feedback about the RN's progress? 1 Nurse manager 2 Healthcare provider 3 Patient care associate 4 Cross-trained technician

1 The nurse manager, who is the delegator for the healthcare team, will be asked to provide feedback about the progress of the RN in the rehabilitation center. The healthcare provider will not be asked to provide feedback about the RN's progress. The patient care associate and cross-trained technician also do not provide feedback about the RN because they do not supervise the RN.

The nurse is caring for a client with chronic pain who just had surgery and is receiving patient-controlled analgesia (PCA) morphine. The client is in severe pain, with a rating of 10/10, despite receiving the maximum ordered dose. The nurse calls the health care provider, saying that the client is still having pain and recommending a higher PCA dose. Which nursing role is being implemented in this situation? 1. Advocate 2. Caregiver 3. Educator 4. Manager

1 The role of the nurse as advocate is to protect the rights of the client, including the right to adequate pain control. The nurse acting as advocate speaks up for clients when they cannot easily speak for themselves. (Option 2) In the role of caregiver, the nurse promotes healing and well-being by helping the client and family set and achieve goals through the nursing process. (Option 3) In the role of educator, the nurse helps the client and family learn about topics relevant to their health. (Option 4) In the role of manager, the nurse coordinates the care of the client among different members of the interdisciplinary team and across care settings. Educational objective: An important nursing role is client advocacy, which involves speaking up for clients to protect their rights and improve their health outcomes and experiences.

After caring for a client who underwent a cesarean section, the delegatee failed to report results and findings of vital signs to the delegator due to improper understanding. Which action of the delegator is the reason for the delegatee's failure? 1 Unclear directions to the delegatee 2 Inappropriate selection of delegatee 3 Inappropriate feedback related to task 4 Inappropriate supervision by the delegator

1 Unclear directions to the delegatee by the delegator may result in failure to report results and findings of vitals by the delegatee. As per the rights of delegation, the delegator may not assign the task to an inappropriate delegatee. Inappropriate feedback related to the task may be due to an information delay that may cause adverse effects in the client. Inappropriate supervision by the delegator may lead to unexpected outcomes.

A client with a 10-year history of methadone use for chronic leg pain is being treated with azithromycin for pneumonia. On the third hospital day, both medications are discontinued as the QT interval on EKG has lengthened, increasing arrhythmia risk. The client wants to be discharged against medical advice to return home and take the client's own medications to prevent going into withdrawal without the methadone. Which is the most appropriate nursing response? 1. "I will ask the HCP to come talk with us so that we can develop a plan to prevent withdrawal while reducing your risk of heart problems." 2. "I will talk with the HCP about your concerns, but in the meantime it's important that you stay here." 3. "It's important that you stay in the hospital so that we can treat you quickly if you have problems." 4. "You have the right to make your own decisions, but you are at high risk of having heart problems if you go home right now."

1 When clients are hospitalized, they lose control of many things, including their medication management. This loss of control can be frightening for the client, especially one who has had control of medications for many years. This client, who has a decade of experience taking methadone for chronic pain, is afraid that suddenly stopping this medication may precipitate withdrawal. The client is trying to regain control and avoid this problem by leaving the hospital against medical advice. However, the client remains at risk of life-threatening arrhythmias. Therefore, the nurse should promote negotiation between the client and HCP to develop a plan of care that will address the concerns of each. The plan should advocate for the client to ensure that the concerns are addressed. Care planning should be a collaborative, shared process informed by the knowledge and preferences of the client and evidence-based recommendations by the HCP that are appropriate to the situation. (Option 2) This response is based on the idea of the nurse and HCP being in control, but it fails to include the client in the decision-making team. (Option 3) This statement provides a rationale for the client to remain in the hospital, but it does not address the client's concerns about going into withdrawal. (Option 4) This response is based on the idea of client autonomy, but it does not propose a solution to the problem. Educational objective: A plan of care should be developed collaboratively, informed by the client's knowledge, beliefs, and preferences, and the expertise and evidence-based recommendations of HCPs.

Which component of delegation is defined as the ability to perform duties in a specific role? 1 Authority 2 Supervision 3 Responsibility 4 Accountability

1 Authority is the ability to perform duties in a specific role. Supervision is the provision of guidance and oversight of a delegated task. Responsibility refers to reliability, dependability, and obligation to accomplish work. Accountability involves determining whether the actions are appropriate and providing a detailed explanation of what has occurred.

Which task followed by the executive nurse is a leadership development task? 1 Maintaining balance 2 Having a clear vision 3 Listening to constituents 4 Maintaining a positive attitude

1 Having a clear vision and seeing beyond where one is and where one is going is a leadership development task. Maintaining balance, listening to constituents, and maintaining a positive attitude are rules for a leader.

A nurse manager proposes to change a protocol regarding the care of postoperative clients. However, the nursing staff openly expresses their resistance to the change and wants to continue with the original protocol. Which behavioral pattern in response to change best describes the nursing staff? 1 Laggards 2 Rejectors 3 Early majority 4 Late majority

1 Laggards value their traditional practices and may openly express their resistance to the change. Rejectors actively resist the change and may use sabotage to avoid the change. An early majority may prefer a conventional approach but accept the change gradually. A late majority is openly resistant to change and does not accept the change until all others have accepted it.

Which type of behavior by the registered nurse results in a low-to-stable level of commitment? 1 Punitive 2 Charismatic 3 Inspirational 4 Intellectual stimulation

1 Punitive behavior is a type of transactional leadership behavior that results in a low-to-stable level of commitment as an organizational outcome. Charismatic, inspirational, and intellectual stimulation are transformational leadership behaviors that result in increased commitment as an organizational outcome.

The registered nurse (RN) is teaching a student nurse about the qualities of a transactional leader. Which statement made by the student indicates effective learning? 1 "I will penalize team members for poor performance." 2 "I will motivate employees by articulating an inspirational vision." 3 "I will encourage employees to transcend their own self-interests." 4 "I will challenge the process that involves new solutions to old problems."

1 "I will penalize team members for poor performance." A transactional leader is responsible for penalizing a nurse executive for poor performance. This type of leader influences followers to fulfill their contracts or get penalized. Motivating employees by articulating an inspirational vision is the behavior of a transformational leader. Encouraging employees to transcend their own self-interests to increase job satisfaction is the behavior of a transformational leader. Challenging the process that involves new solutions to old problems is the behavior of a transformational leader.

Which question does the registered nurse recognize as related to the right of circumstance when delegating? 1 "Is the delegation appropriate to the situation?" 2 "Is the task within the delegatee's scope of practice?" 3 "Is the prospective delegate a willing and able employee?" 4 "Is the delegator able to monitor and evaluate the client appropriately?"

1 "Is the delegation appropriate to the situation?" The question, "Is the delegation appropriate to the situation?" is related to right circumstance. The question, "Is the task within the delegatee's scope of practice?" is related to right task. The question, "Is the prospective delegate a willing and able employee?" is related to right person. The question, "Is the delegator able to monitor and evaluate the client appropriately?" is related to right supervision.

A nurse leader, along with the team, is caring for a client who is scheduled for colonoscopy. Which delegated task requires the leader's supervision? 1 Assisting the client with an enema 2 Assisting the client with bathing 3 Assisting the client with feeding 4 Assisting the client with ambulating

1 Assisting the client with an enema A client who is scheduled for colonoscopy requires having the gastrointestinal tract emptied, mainly the colon. Before undergoing this procedure the client is assisted with an enema and other laxatives to expel all bowel contents present in the gastrointestinal tract. A nurse leader should supervise this procedure, as an inadequately emptied colon may cause complications. Assisting the client with bathing, feeding, and ambulating are easy tasks and may not require supervision.

Who acts as a delegator in the absence of the registered nurse? 1 Charge nurse 2 Patient care associate 3 Licensed practical nurse 4 Unlicensed nursing personnel

1 Charge nurse The charge nurses act as a delegator on the basis of knowledge and experience in clinical settings. In the absence of a registered nurse, the charge nurse usually delegates the tasks. A patient care associate assists and monitors vital signs. Licensed practical nurses and unlicensed nursing personnel are considered delegatees due to insufficient experience and training.

Which of these clients is most appropriate for delegation to a cross-trained technician? 1 Client A - lung cancer; lobectomy treatment; postop care is providing respiratory therapy 2 Client B - CAD; cardiac bypass treatment; postop care is continuous ECG monitoring 3 Client C - dysmenorrhea; hysterectomy treatment; postop care is monitoring of vitals 4 Client D - fractured pelvis; observation as treatment; postop care is teaching leg exercises

1 Client A A cross-trained technician, when paired with a registered nurse (RN), may perform respiratory therapy, phlebotomy, and electrocardiography. Care of client A can be safely delegated to a cross-trained technician. Though the cross-trained technician can record the ECG, he or she may not monitor the continuous ECG. Monitoring of vital signs is also the role of the RN. Leg exercises may be taught by the RN or by a physician. The other licensed assistive personnel may be delegated the task of reinforcing the teaching, but this is not an appropriate role for the cross-trained technician.

What aspect of total client care needs to be taken into consideration if used on a 24-bed care area? 1 Cost of nurses 2 Training needs 3 Documentation 4 Time commitment

1 Cost of nurses Since total client care is provided by a registered nurse (RN) it can be very expensive and not realistic to use a highly skilled and extremely knowledgeable professional nurse to provide all the care required in a unit of 24 clients. There are no particular issues with training, documentation, or time commitment with total client care.

The leader encounters a situation where the nursing outcomes are unpredictable. Which attributes or approaches should the leader utilize to navigate this situation? Select all that apply. 1 Creativity 2 Experience 3 Enthusiasm 4 Group process 5 Independent decision making

1 Creativity, 2 Experience, 4 Group process A situation with unpredictable results requires creativity, experience, and group process. Enthusiasm is an important quality of a follower but it would be a secondary factor to be considered in this situation. Independent decision making would not be appropriate to this situation. Rather, the leader should seek expert opinion and involve key stakeholders.

The registered nurse (RN) delegates the collection of respiratory rate data to a licensed practical nurse (LPN) for a client who is experiencing severe dehydration and whose condition is unstable. The LPN reports the data to the RN. The RN rechecks the data and finds that the report no longer reflects the client's current condition. Which characteristic of communication has interfered with the delegation process? 1 Information decay 2 Information salience 3 Confidence in abilities 4 Synergy between team members

1 Information decay Information decay can occur in a rapidly changing situation when reported information is no longer relevant to a client's condition. Information salience describes the different ways individuals from different backgrounds might assess the quality, meaning, and clarity of certain information. Trust is developed when there is confidence in the abilities and capabilities of the team members. Healthy relationships among members of the health care team promote synergy between the team members.

Which statement describes the practice partnership model of care delivery? 1 Nurse and unlicensed assistive personnel work the same shifts and days. 2 Nurse provides unlicensed assistive personnel (UAP) report for a group of clients. 3 Unlicensed assistive personnel (UAP) perform the same task for a group of clients. 4 Unlicensed assistive personnel (UAP) measure vital signs and provide morning care.

1 Nurse and unlicensed assistive personnel work the same shifts and days. In the practice partnership model of providing client care, a registered nurse (RN) is paired with a technical assistant who works with the RN consistently, which means they are scheduled to work the same shifts and days. The nurse providing a report for a group of clients would be an action within team nursing. Unlicensed assistive personnel (UAP) performing the same task for a group of clients would be seen in functional nursing. Unlicensed assistive personnel (UAP) measuring vital signs and performing morning care would be a part of team or functional nursing. In the practice partnership model, the nurse and UAP would measure vital signs and perform care together.

Before assigning a task, the registered nurse makes sure that the prospective delegatee is willing to complete the task. Which delegation right does this situation reference? 1 Person 2 Supervision 3 Circumstance 4 Communication

1 Person Ensuring that the prospective delegatee is willing to complete the task refers to the delegation right person. Knowing whether the delegator is able to monitor and evaluate the client appropriately refers to the delegation right supervision. Ensuring whether the equipment and resources are available to complete the delegation process refers to circumstance. Ensuring whether the delegator and delegatee understand a common work-related language refers to the delegation right communication.

Which responsibility of the nurse manager differs from the responsibilities of a nurse leader? 1 Planning the budget 2 Motivating the team 3 Resolving the conflicts 4 Penalizing for poor performance

1 Planning the budget Budgeting is the responsibility of a nurse manager. The role of nurse leader is to motivate the nursing subordinates. Resolving conflicts is the responsibility of both the nurse leader and the nurse manager. Penalizing for poor performance is a behavior of transactional leadership.

The nurse is caring for a pregnant client with hypertension. Which client care tasks are most suitable to be delegated to the patient care associate (PCA)? 1 Recording the vital signs 2 Monitoring the blood pressure 3 Administering intravenous fluids 4 Administering antihypertensive medications

1 Recording the vital signs A patient care associate (PCA) is an unlicensed assistive personnel whose scope of practice is very limited. A PCA can be delegated the task of recording the vital signs as communicated by the delegator even if the condition of the client is acute or unstable. In stable clients, the PCA may be instructed to monitor the blood pressure, but in this acute condition, only the registered nurse (RN) should monitor the blood pressure. Administration of intravenous fluids or medications is out of scope of practice of the PCA. A PCA is not suitable to be delegated the task of administering any medication to a client.

Which actions indicate that a manager is helping staff adjust to functional nursing? Select all that apply. 1 Staff meeting conducted every Tuesday at 1 pm 2 Charge nurse assigned to analyze client outcomes 3 Staff assigned to make out daily assignments sheet 4 Assigned tasks rotated among same category of care providers 5 Staff nurses assigned to review research studies for evidence-based care

1 Staff meeting conducted every Tuesday at 1 pm 4 Assigned tasks rotated among same category of care providers Since functional nursing can appear to be autocratic, the nurse manager can improve the staff's lack of independence by conducting frequent staff meetings and rotating assigned tasks among members of the same category of care providers. The nurse manager should analyze client outcomes. The charge nurse makes out daily assignment sheets. Reviewing research studies is not a client-focused task in functional nursing. The functional model of nursing is a method of providing client care by which each licensed and unlicensed staff member performs specific tasks for a large group of clients.

What is the priority nursing care for a client who is prescribed hydroxychloroquine (Plaquenil)? 1 Teaching the client to report blurred vision 2 Teaching the client to report signs of infection 3 Teaching the client to report shortness of breath 4 Teaching the client to report stomach discomfort

1 Teaching the client to report blurred vision Plaquenil is a hydroxychloroquine used to treat rheumatoid arthritis. The adverse effect of Plaquenil is retinal damage; therefore, a client on Plaquenil is taught to report blurred vision. A client on steroids is taught to report signs of infection. A client on infliximab is taught to report shortness of breath. Plaquenil causes mild stomach discomfort, which is normal and is not related to priority nursing care.

Which point should the nurse exclude when developing strategies to project a powerful image? 1 Using authoritative language 2 Maintaining good body posture 3 Making good eye contact with clients 4 Ensuring that clothing and hair are appropriate to the situation

1 Using authoritative language The nurse should treat people with courtesy and respect. The use of authoritative language does not help in developing a powerful image. The nurse should also maintain good body posture because it conveys self-confidence. Good eye contact helps in developing trust. Appropriate dress and hair help convey a powerful image.

The charge nurse is reviewing clients' medical records on the cardiovascular care unit. Which client care outcomes are appropriate? Select all that apply. 1. Client receiving a continuous heparin infusion for a deep venous thrombosis remains free of petechiae or purpura 2. Client who had a carotid endarterectomy maintains a heart rate <100/min and blood pressure >90/60 mm Hg with no neurological changes 3. Client who had a percutaneous coronary intervention maintains a chest pain level of <4 on a scale of 0-10 while at rest 4. Client with hypertension receiving IV furosemide remains free from muscle cramping in the extremities 5. Client with peripheral arterial disease following a femoral-popliteal angioplasty remains free of leg pain during ambulation

1, 2, 4, 5 Clients receiving IV heparin should maintain therapeutic clotting times, avoid developing embolic events, and remain free from signs of heparin-induced thrombocytopenia (eg, petechiae, purpura) (Option 1). Clients having undergone a carotid endarterectomy, a surgical procedure removing plaque from carotid arteries, would be expected to show no evidence of hemorrhage (eg, hypotension, tachycardia) or neurological impairment (eg, decreased level of consciousness, altered mental status) (Option 2). Clients receiving IV furosemide, a loop diuretic, should maintain adequate blood pressure and avoid developing symptoms of electrolyte imbalance (eg, muscle weakness, cramps, cardiac arrhythmia) (Option 4). A femoral-popliteal angioplasty is a surgical procedure to restore perfusion to the legs of clients with peripheral arterial disease. After the procedure, the client should be able to ambulate without evidence of extremity ischemia (eg, leg pain) (Option 5). (Option 3) A percutaneous coronary intervention (PCI) is a procedure used to restore coronary perfusion to prevent or treat ischemia or infarction. Clients having undergone a PCI would be expected to have no chest pain at rest. Chest pain at rest indicates myocardial ischemia. Educational objective: Clients receiving heparin should remain free from heparin-induced thrombocytopenia. After carotid endarterectomy, clients should remain free from hemorrhage and neurological impairment. Those receiving loop diuretics should maintain electrolytes within normal limits. After a femoral-popliteal angioplasty, clients should be able to ambulate without leg pain. They should have no chest pain at rest after a percutaneous coronary intervention.

There has been a major community disaster. Stable clients need to be discharged to make more beds available for the victims. Which clients could be discharged safely? Select all that apply. 1. Diagnosed with endocarditis on antibiotics with a peripherally inserted central catheter (PICC) line 2. History of multiple sclerosis with ataxia and diplopia 3. One day postoperative from a hemicolectomy 4. Reporting abdominal pain with coffee ground emesis 5. Taking warfarin with prothrombin time/International Normalized Ratio of 2x control value

1, 2, 5 Ataxia and diplopia are expected signs/symptoms of multiple sclerosis. Two times the control value demonstrates that warfarin has reached a therapeutic level. The long-term antibiotic course (and follow-up lab work) can continue at home through the PICC line (Options 1, 2, and 5). (Option 3) Large intestine peristalsis does not return for up to 3-5 days. The client cannot be discharged until able to tolerate oral intake with normal elimination. The client has to at least be passing flatus. (Option 4) Coffee ground emesis indicates upper gastrointestinal bleeding. The etiology and treatment need to be determined before the client is discharged. Educational objective: Those who are stable for discharge include the client with multiple sclerosis with ataxia and diplopia, the client on warfarin (Coumadin) that has reached the therapeutic effect, and the client with a PICC line for a long-term antibiotic course.

Which characteristics of a licensed practical nurse should be considered before delegation of tasks? Select all that apply. 1 Critical thinking 2 Effective client care 3 Diagnostic reasoning 4 Synthesizing information 5 Accountability of assigned task

1, 3, 4 Critical thinking, diagnostic reasoning, and the ability to synthesize information from various sources are three factors that are characteristics of the licensed practical nurse that should be considered before delegating tasks. Planning effective client care is the role of a registered nurse. Accountability for an assigned task is the basic responsibility of a nurse.

The nursing manager wants to implement transformational leadership qualities. Which actions best describe this style of leadership? Select all that apply. 1 Providing intellectual stimulation 2 Intervening only when problems exist 3 Performing experiments with system redesign 4 Using motivator factors to inspire work performance 5 Monitoring performance and taking action to correct

1, 3, 4 Transformational leaders provide intellectual stimulation and encourage follower creativity. They experiment with systems redesign, empower staff, create enthusiasm for practice, and promote scholarship of practice in client care. They use motivator factors liberally to inspire work performance. Intervening only when problems exist is a behavior related to transactional leadership. Monitoring performance and taking corrective action is a transactional leadership quality.

A nurse educator is developing materials for a hospital-wide campaign about zero tolerance for lateral violence and bullying among staff. Which actions will the nurse educator include in teaching about what staff members should do if they experience workplace violence? Select all that apply. 1. Document the interactions with the bully 2. Ignore the bully's comments, remarks, and allegations 3. Observe interactions between the bully and other colleagues 4. Report the violent incidents to the hospital administrator 5. Tell the bully you will not tolerate the unprofessional behavior

1, 3, 5 Lateral violence (also known as horizontal violence) can be defined as acts of aggression carried out by a co-worker against another co-worker and designed to control, diminish, or devalue a colleague. These behaviors usually take the form of verbal abuse such as name-calling, unwarranted criticism, intimidation, and blaming. However, other acts, such as refusing to help someone, sabotage, exclusion, and unfair assignments, also fall under the category of lateral violence. Violence in the workplace should not be tolerated or ignored by either staff or management. Actions that staff members can take if they become victims of lateral violence include: - Documenting and keeping a file of all incidents (Option 1) - Reporting the incidents to the immediate supervisor - Letting the bully know that the behavior will not be tolerated (Option 5) - Observing interactions between the bully and other colleagues (may validate the victim's experiences and serve as a source of support) (Option 3) - Seek support from within the facility or from an external source (Option 2) Ignoring acts of lateral violence will perpetuate the bullying. (Option 4) The chain of command should be followed when reporting incidents of lateral violence. If the immediate supervisor takes no action, the employee can move up the chain. Educational objective: Lateral violence in the workplace (acts of aggression by an employee toward another employee) should not be tolerated or ignored. Victims can take action against bullying, including documenting and reporting incidents, standing up to the bully in a professional way, and seeking support.

The nurse leader arranges a meeting with the staff members to learn how satisfied they are with their jobs. Which approach by the nurse leader would outline effective leadership? Select all that apply. 1 Seeking feedback from the team 2 Allotting work schedules to the team 3 Penalizing the team for poor performance 4 Motivating the team to reach the desired goals 5 Asking the team to participate in decision making

1, 4, 5 Seeking feedback from the team will help the team members in reaching the organizational goals and a shared vision. The most important leadership quality is to motivate the team members in such a way that they are satisfied with their jobs and helped in reaching their goals. Asking team members to participate in decision making will improve their job satisfaction. Allotting the work schedules and penalizing team members for poor performance are the duties of a nurse manager.

Which statements involve acceptable use of an abbreviation, symbol, or dose designation in documentation? Select all that apply. 1. "2 cm × 3 cm × 1 cm stage II decubitus noted on left shin." 2. "4.0 u SSRI administered to cover capillary glucose of 160 mg/dL." 3. "Dose of .5 mg hydromorphone administered and the client feels 'better.'" 4. "Maalox 5 mL PO administered pc as requested for c/o heartburn." 5. "Spouse voiced understanding of home urinary catheterization QID."

1, 4, 5 The Joint Commission (2004) and Institute for Safe Medication Practices prohibit error-prone or "dangerous" abbreviations, descriptions of symptoms, and dose designations in medical documentation. "Cm" (centimeters) and "II" (2) (eg, decubitus staging) are acceptable abbreviations/notations (Option 1). The abbreviations "ac" (before meals), "pc" (after meals), and "c/o" (complains of) are acceptable (Option 4). "QID" (4 times a day) is acceptable. Abbreviations that are not acceptable include "qd" (daily) and "q1d" (daily), which can be mistaken for "qid" (4 times a day), and "qod" (every other day), which can be mistaken for "qd" (daily) (Option 5). (Option 2) A trailing zero after the decimal point is not acceptable as it could be interpreted as 40 instead of 4 if the decimal point is not noted. The use of "u" for unit is not acceptable as it can be mistaken for the number 0 or 4 (eg, 4u seen as 40). "SSRI" (sliding-scale regular insulin) is not acceptable to indicate insulin as it can be mistaken for selective serotonin reuptake inhibitor. "Mg" for milligrams is acceptable. (Option 3) A zero must precede the decimal dose. If the decimal point is missed, ".5" could be mistaken for 5 mg. Educational objective: Acceptable abbreviations include "ac," "pc," "QID," and "cm." Unacceptable abbreviations include "qd," "q1d," and "qod"; "SSRI" for insulin; and "u" for units. There must be a zero before a decimal dose and no trailing zero after a decimal point.

Which are considered as nurse competencies within the synergy model of care delivery? Select all that apply. 1 Collaboration 2 Care planning 3 Communication 4 Clinical judgment 5 Cultural competency

1, 4, 5 The Synergy Model describes eight nurse competencies, which include collaboration, clinical judgment, advocacy and moral agency, caring practices, facilitation of learning, systems thinking, response to diversity (cultural competency), and clinical inquiry. Care planning and communication are skills that every nurse should demonstrate regardless of the care delivery system being implemented.

According to the three-tiered triage system, which client requires urgent treatment? Select all that apply. 1 A client with renal colic 2 A client with strains and sprains 3 A client with respiratory distress 4 A client with severe abdominal pain 5 A client with multiple displaced fractures

1, 4, 5 According to the three-tiered triage system, renal colic, severe abdominal pain, and multiple displaced fractures require urgent treatment. Strains and sprains require nonurgent treatment and respiratory distress requires emergent treatment.

Which are the highest priorities when conducting a primary client survey during the emergency assessment? Select all that apply. 1 Airway 2 Disability 3 Breathing 4 Circulation 5 Cervical spine

1, 5 Airway and stabilization of the cervical spine are the top priorities when conducting a primary client survey during the emergency assessment. The nurse will then focus on breathing, circulation, and disability.

The nurse leader actively participates in the quality improvement process. Arrange the steps in a sequence that the nurse leader will follow while implementing the quality improvement process? 1. Select a plan to meet the outcomes. 2. Collect the data for evaluating the plan. 3. Establish the outcomes and quality indicators. 4. Identify the important needs of the consumer. 5. Collect the data on the current status of the service. 6. Collaborate with the interprofessional team to review needs.

1. Identify the important needs of the consumer. 2. Collaborate with the interprofessional team to review needs. 3. Collect the data on the current status of the service. 4. Establish the outcomes and quality indicators. 5. Select a plan to meet the outcomes. 6. Collect the data for evaluating the plan. The first step of the quality improvement process involves identifying the needs that are important to the consumer of healthcare services. Collaborating with the interprofessional team to review the needs that are identified is done to avoid bias. The next step is confirming the needs and collecting the data on the current position of the service that helps in the improvised plan. This is followed by implementing the process and establishing the outcomes for which quality indicators are required. Then the next step is to select and implement the plan to meet the outcomes and objectives of quality improvement. Evaluating the plan is the last step and is necessary for quality improvement.

The nurse is teaching a client how to teach the supraglottic method of swallowing. Arrange the prioritizing order of statements by the nurse for teaching the client. 1. "Swallow twice." 2. "Hold your breath." 3. "Clear your throat." 4. "Take a deep breath." 5. "Place yourself in an upright position." 6. "Place a half to 1 teaspoon of food into your mouth."

1."Place yourself in an upright position." 2."Clear your throat." 3."Take a deep breath." 4."Place a half to 1 teaspoon of food into your mouth." 5."Hold your breath." 6."Swallow twice." The order of steps to be followed in instructing the client in the supraglottic method of swallowing is first to place the client in an upright position, secondly to tell the client to clear the throat, and then to take a deep breath. The fourth step is for the client to place a half or one teaspoon of food into the mouth. The client should hold his or her breath and then swallow twice. This method exaggerates the normal protective mechanisms with cessation of respiration during the swallow.

The nurse is performing resuscitation interventions for airway, breathing, and circulation as part of a primary survey in a client. Which order of actions should the nurse follow for this client? 1. Use direct pressure for external bleeding. 2. Prepare for chest decompression if needed. 3. Assess breath sounds and respiratory effort. 4. Establish airway by positioning, suctioning, and oxygen as needed. 5. Maintain vascular access using a large-bore catheter.

1.Establish airway by positioning, suctioning, and oxygen as needed. 2.Assess breath sounds and respiratory effort. 3.Prepare for chest decompression if needed. 4.Maintain vascular access using a large-bore catheter. 5.Use direct pressure for external bleeding. The primary survey includes assessment of airway/cervical spine, breathing, circulation, disability, and exposure. First, the nurse should establish airway patency by positioning, suctioning, and providing oxygen as needed. Assess breath sounds and respiratory effort and provide chest decompression if needed in order to assess breathing. Maintain vascular access using a large-bore catheter and use direct pressure for any external bleeding.

Which order of steps would the nurse teach the client to follow while performing expansion breathing? 1. Place hands on each side of lower ribcage, just above the waist. 2. Sit in an upright position with knees slightly bent. 3. Exhale, first moving the chest and then lower ribs inward while gently squeezing the ribcage, forcing air out of the base of lungs. 4. Take a deep breath through your nose, using shoulder muscles to expand your lower ribcage outward during inhalation.

1.Sit in an upright position with knees slightly bent. 2.Place hands on each side of lower ribcage, just above the waist. 3.Take a deep breath through your nose, using shoulder muscles to expand your lower ribcage outward during inhalation. 4.Exhale, first moving the chest and then lower ribs inward while gently squeezing the ribcage, forcing air out of the base of lungs. While performing expansion breathing, the client should sit in an upright position with slightly bent knees because it decreases tension on the abdominal muscles and respiratory resistance and discomfort. Then the client should place hands on each side of the lower ribcage, just above the waist. A deep breath through the nose is taken, using shoulder muscles to expand the lower ribcage outward during inhalation. The client then exhales by first moving the chest and then lower ribs inward while gently squeezing the ribcage, forcing air out of the lungs.

After receiving the shift report, the nurse should assess which infant first? 1. An infant born 6 hours ago after 38 weeks gestation who has a respiratory rate of 52/min 2. An infant born 12 hours ago who is jittery and has a blood glucose level of 40 mg/dL (2.2 mmol/L) 3. An infant with bilateral crackles who was delivered vaginally 30 minutes ago 4. An infant wrapped in a warm blanket 15 minutes ago due to a temperature of 97.7 F (36.5 C)

2 A normal blood glucose range for an infant is 40-60 mg/dL (2.2-3.3 mmol/L) within the first 24 hours after delivery. A blood glucose level <40 mg/dL (2.2 mmol/L) indicates hypoglycemia. Symptoms of hypoglycemia include jitters, cyanosis, tremors, pallor, poor feeding, retractions, lethargy, low oxygen saturation, and seizures. This infant with borderline-low glucose level is symptomatic and should be assessed first. (Option 1) A normal respiratory rate for an infant is 30-60/min. This infant is currently stable. (Option 3) It is normal to auscultate crackles in an infant during the first hour of life. This is because fluid is still being pushed out of and absorbed by the lungs. This infant is currently stable. (Option 4) A normal temperature range for an infant is 97.7-99.7 F (36.5-37.6 C). This infant is currently stable. Educational objective: The nurse should monitor infants for hypoglycemia by assessing for symptoms and monitoring the blood glucose level. A blood glucose level <40 mg/dL (2.2 mmol/L) indicates hypoglycemia and should be treated immediately by feeding or administering a glucose bolus.

The charge nurse must assign a semi-private room to a client with diabetes mellitus admitted for IV antibiotic therapy to treat leg cellulitis. Which of the 4 room assignments is the best option for this client? 1. Room 1: Client 1 day postoperative laparoscopic cholecystectomy who is awaiting discharge 2. Room 2: Client with dementia and urinary incontinence wearing an external urine collection device 3. Room 3: Client with history of splenectomy 15 years ago, now admitted for pulmonary embolism 4. Room 4: Client with lupus nephritis who is prescribed treatment with azathioprine

2 Cellulitis is a common skin bacterial infection that is usually treated with IV antibiotics in clients with diabetes mellitus. Room 2 is the best assignment option for this client with cellulitis. The client with dementia and urinary incontinence who has an external urinary condom catheter is the least susceptible to infection compared to those in rooms 1, 3, and 4. (Option 1) The client who is 1 day postoperative laparoscopic cholecystectomy (surgical procedure with small incisions) is at increased risk for infection. The client with cellulitis should not be placed in room 1. (Option 3) Although this client has pulmonary embolism, the history of prior splenectomy leads to a very high lifelong risk of rapid sepsis. Splenectomy clients need vaccination against encapsulated organisms (eg, pneumococcus, meningococcus, and Haemophilus influenzae type B). Even a low-grade fever should be taken seriously in these clients. The client with cellulitis should not be placed in room 3. (Option 4) Lupus nephritis is a serious renal complication of systemic lupus erythematosus (SLE), an inflammatory autoimmune disease that can lead to end-stage kidney disease. The systemic disease and the immunosuppressant (azathioprine [Imuran]) prescribed to slow its progression increase infection risk. The client with cellulitis should not be placed in room 4. Educational objective: A client with an infection should not be assigned to a semi-private room with a client who had surgery or is immunocompromised and receiving immunosuppressants as these clients are highly susceptible to infection. Post-splenectomy clients are also at lifelong risk for rapid sepsis.

Which statement about the delegatee condition is true regarding "selling"? 1 The delegatee has established relationships and expertise. 2 The delegatee has ongoing relationships; however, the task is new. 3 The delegatee has limited knowledge and ability to perform the task. 4 The delegatee has willingness and ability, but the relationship is new.

2 "Selling" in context with delegation represents an ongoing relationship with a new task being delegated. "Participating" in the context of delegation includes willingness and ability to perform a task with a new relationship. "Telling" in the context of delegation includes limited knowledge and ability to perform the task by the delegatee. "Delegating" in context with delegation represents established relationships and expertise.

The registered nurse (RN) is getting ready to leave the client care unit for a lunch break. The RN asks the LPN to take care of a client during the lunch break. Which concept is emphasized in this situation? 1 Leadership 2 Delegation 3 Supervision 4 Assignment

2 Delegation is the concept of a transfer of authority between two people to perform some type of task, e.g., between an RN and licensed staff. Leadership is the action of leading a group or an organization. Supervision is defined as the active process of directing, guiding, and influencing the outcome of an individual's performance. Assignment is the transfer of both accountability and responsibility.

The nurse manager orders the direct care nurse to assist in providing tertiary care to a client. Which type of treatment would the direct care nurse be involved in? 1 Chronic care 2 Rehabilitation 3 Health maintenance 4 Prevention of disease complications

2 Tertiary care includes rehabilitation. Chronic care and health maintenance are considered primary care, and prevention of disease complications is classified as secondary care.

The healthcare team is caring for a client with neutropenia. Which task is delegated to unlicensed assistive personnel? 1 Administering antibiotics 2 Assisting with personal hygiene 3 Monitoring for signs and symptoms of infection 4 Teaching the client and caregivers about how to avoid infection

2 Unlicensed assistive personnel assisting with the client's personal hygiene. The licensed practical nurse administers antibiotics. Monitoring for signs and symptoms is performed by the licensed practical nurse. The registered nurse teaches the client and caregivers how to avoid infection.

A client's serum potassium level has increased to 5.8 mEq/L (5.8 mmol/L). What action should the nurse implement first? 1 Call the laboratory to repeat the test. 2 Take vital signs and notify the healthcare provider. 3 Inform the cardiac arrest team to place them on alert. 4 Take an electrocardiogram and have lidocaine available.

2 Vital signs monitor cardiorespiratory status; hyperkalemia causes cardiac dysrhythmias. The healthcare provider should be notified because medical intervention may be necessary. A repeat laboratory test will take time and probably reaffirm the original results; the client needs immediate attention. The cardiac arrest team is always on alert and will respond when called for a cardiac arrest. Taking an electrocardiogram and having lidocaine available are insufficient interventions.

Which comment by a team leader exemplifies team nursing the way it was intended? 1 "(Unlicensed assistive personnel), I need you to measure all vital signs." 2 "(Registered Nurse 1), check all of the intravenous (IV) sites and give the IV medications." 3 "(Licensed Practical Nurse), you pass all of the oral medications to the clients today." 4 "(Registered Nurse 2), Mr. Jones has a foot wound, two IV meds, and needs morning insulin."

2 "(Registered Nurse 1), check all of the intravenous (IV) sites and give the IV medications." The team leader must have excellent communication skills, delegation, and effective decision-making abilities to provide a working "team" environment for members. When the team leader is not prepared for this role, the team method becomes a miniature version of the functional method. The assignment given to RN 1 takes the nurse's experience and skills into consideration and exemplifies team nursing. The statements made to the UAP, LPN, and RN 2 exemplify functional nursing.

Which statement of the nurse leader reflects the actions suggested by the Joint Commission for disruptive behavior by a direct care nurse? 1 "This is the last warning for you." 2 "You are terminated, effective now." 3 "This will affect your assessment markings." 4 "You should not behave this way because you are very talented."

2 "You are terminated, effective now." According to the actions suggested by the Joint Commission, there is "zero" tolerance for an intimidating and/or disruptive behavior. The action to be taken for this type of behavior should be suspension, termination, loss of clinical privileges, and reporting to professional licensure bodies. The actions suggested by the Joint Commission do not include giving last a warning. The nurse who has behaved disruptively is not eligible for any assessment. By saying, "You should not behave this way because as you are very talented," the actions suggested by Joint Commission are not being carried out.

A nurse is assisting another registered nurse in the intensive care unit who is caring for a client with uncontrolled blood pressure. Which action on the part of the nurse indicates "offering" during delegation decisions? 1 "Did you check the client's blood pressure this morning?" 2 "You can use this stethoscope and sphygmomanometer to take the blood pressure." 3 "Would you fetch me the client's laboratory reports? I'll record the blood pressure." 4 "How is the client doing this morning? Let me look at yesterday's blood pressure report."

2 "You can use this stethoscope and sphygmomanometer to take the blood pressure." Offering involves making a suggestion to facilitate the achievement of a desirable client care outcome. The statement, "You can use this stethoscope and sphygmomanometer to take the blood pressure," is an example of offering. Asking begins with questions related to the problem or issue regarding client care. "Did you check the client's blood pressure this morning?" is an example of asking. Doing involves demonstrating the specific task or behavior needed to improve client care. "Would you fetch me the client's laboratory reports? I'll record the blood pressure," is an example of doing. "How is the client doing this morning? Let me look at yesterday's blood pressure report," is an example of supervision.

The registered nurse (RN) and unlicensed assistive personnel (UAP) have been working together for two years. Which statement made by the RN would be appropriate after delegating a task to the UAP? 1 "Let me tell you how to do this task." 2 "You know what to do and when to report." 3 "Please tell me how you are going to perform this procedure." 4 "It is important that you check the client's temperature every hour."

2 "You know what to do and when to report." According to Hersey's model of situational leadership, the delegator should act according to the situation. When there is an established relationship between the delegator and the delegatee, little guidance needs to be provided to the delegatee. When the relationship is new and a new task is delegated, an explanation is required about how to perform the task. When the delegatee is newly assigned, then the delegator checks the ability of the delegatee by asking how to perform the procedure. When the relationship between the delegatee and delegator is new, and it is for a limited period, then the delegator just informs the delegatee what is to be done.

The chief operating officer (COO) discusses the performance of a newly appointed leader with the nursing staff. The team communicates that the practical approach of the leader fulfills the quantum theory. What type of work environment does the COO understand that the leader creates at the workplace? 1 An environment that promotes hygienic factors 2 An environment that is rapidly changing and dynamic 3 An environment that provides intellectual stimulation 4 An environment that develops trust between followers and leaders

2 An environment that is rapidly changing and dynamic "Quantum theory" lays its foundation on an environment that is rapidly changing. It does not focus on a single event. Hygienic factors in the environment are an aspect of the two-factor theory. The nurse leader, through the transformational theory, provides intellectual stimulation. An environment that develops trust between followers and leaders are aspects of the situational-contingency theory.

The registered nurse is assessing the conditions of four different clients. Which client's care should the registered nurse delegate to an unlicensed assistive personnel (UAP)? 1 Client A - brain injury; acute care 2 Client B - diarrhea; sub-acute care 3 Client C - uncontrolled DM; chronic care 4 Client D - HTN; acute care

2 Client B - diarrhea; sub-acute care Client B is delegated to the UAP since they are assigned to perform hygienic tasks and diarrhea is a sub-acute condition. Brain injury is a serious condition and care cannot be performed by the UAP. Uncontrolled diabetes is a chronic condition, so client C is not delegated to UAP. Hypertension is an acute condition, and client D is not delegated to the UAP.

The nurse leader noticed that the staff nurse recently promoted to the surgical unit is lacking confidence at work and is worried about a pending review by the nursing director. Which source of power is applicable in this situation? 1 Reward power 2 Coercive power 3 Referent power 4 Connection power

2 Coercive power Coercive power stems from a real or perceived fear of another person. Reward power is perceived as being able to provide rewards or favors. Association with a powerful person grants referent power. Association with people who are powerful or who have links to powerful people gains connection power.

The case manager notes that a client's hospitalization is being prolonged for one day because of an unexpected adverse medication reaction. What action should the case manager take? 1 Add the extra day into the pathway. 2 Document the reaction as a variance. 3 Change all uses of the pathway to add the extra day. 4 Prepare documentation to submit to Medicare about the extra day.

2 Document the reaction as a variance. If a client's progress deviates from the normal path, a variance is indicated. A variance is anything that occurs to alter the client's progress through the normal critical path. The reason(s) for the variance should be analyzed and the care revised to meet the needs of the clients. An extra day is not added into the pathway and all uses of the pathway are not changed by adding an extra day based on this single variance. There is no evidence to support that the case manager needs to submit documentation to Medicare about the extra day.

A client reports severe pain 2 days after surgery. After assessing the characteristics of the pain, which initial action should the nurse take next? 1 Encourage rest. 2 Obtain vital signs. 3 Administer the prescribed analgesic. 4 Document the client's pain response.

2 Obtain vital signs. Immediately before administration of an analgesic, an assessment of vital signs is necessary to determine whether any contraindications to the medication exist (e.g., hypotension, respirations ≤12 breaths/min). Pain prevents both psychological and physiologic rest. Before administration of an analgesic, the nurse must check the healthcare provider's prescription, the time of the last administration, and the client's vital signs. A complete assessment, including vital signs, should be done before documenting the client's pain response.

A hospital management team has conducted a randomized controlled trial to decrease the occurrence of ventilator-associated pneumonia. The trial was successful and had positive outcomes. The nurse manager, in collaboration with other hospital management staff, conducted the same trial in another hospital, but the results were different. Which research strategy implementation would the nurse manager consider to be beneficial in preventing dramatic differences in trial results? 1 Evidence-based practice (EBP) 2 Practice-based evidence (PBE) 3 Client-centered outcomes research 4 Comparative effectiveness research (CER)

2 Practice-based evidence (PBE) PBE is a research methodology that helps inform practice. It uses an observational cohort study design that compares clinically relevant interventions, includes diverse study participants, uses heterogeneous practice settings, collects data on a broad range of health outcomes, and includes frontline clinicians in study development. EBP is the integration of the best research evidence with clinical expertise and the client's unique values and circumstances in making decisions about the care of individual clients. Client-centered outcomes researchers conduct research to provide information about the best available evidence to help clients and their providers make decisions that are more informed. CER is the generation and synthesis of evidence that compares the benefits and harms of alternative methods.

A client is admitted to the emergency department following a motor vehicle accident. The client's wounds are extensive. Which healthcare team member is best suited to care for this client in the emergency ward? 1 Charge nurse 2 Registered nurse 3 Licensed practical nurse 4 Unlicensed nursing personnel

2 Registered nurse A registered nurse should be the healthcare team member to care for the client in the emergency department. The charge nurse's role includes making client assignments, scheduling breaks for staff members, and serving as a staff resource person. The licensed practical nurse is involved in fast-track emergency care. Unlicensed nursing personnel perform all hygienic tasks and are not required in the emergency care unit.

A nurse in the postanesthesia care unit (PACU) is providing care to a client who had an abdominal cholecystectomy and observes serosanguineous drainage on the abdominal dressing. What is the next nursing action? 1 Change the dressing. 2 Reinforce the dressing. 3 Replace the tape with Montgomery ties. 4 Support the incision with an abdominal binder.

2 Reinforce the dressing. The nurse should anticipate drainage and reinforce the surgical dressing as needed. Changing a dressing at this time is unnecessary and increases the risk for infection. Montgomery ties are used when frequent dressing changes are anticipated; they are not appropriate at this time. An abdominal binder rarely is prescribed, and it will interfere with assessment of the dressing at this time.

The nurse finds the respiratory rate is 8 breaths per minute in a client who is on intravenous morphine sulfate. What should the nurse do immediately in this situation? 1 Measure other vital signs. 2 Stop administering the medication. 3 Elevate the head of the client's bed. 4 Report to the primary healthcare provider

2 Stop administering the medication. Morphine sulfate is an opioid analgesic and can depress the central nervous system, which results in respiratory depression. A respiratory rate of 8 breaths per minute indicates respiratory depression, and the nurse should stop the medication immediately. The nurse can measure the other vital signs after discontinuing the medication administration. Elevating the head of the client's bed ensures proper breathing. Therefore the nurse should elevate the client's bed after discontinuing the medication. The nurse should report to the primary healthcare provider for an appropriate antidote after stopping the medication administration.

When does delegation become more challenging? Select all that apply. 1 When the client is stable 2 When the client is pregnant 3 When the geographic area is small 4 When the resources are abundant 5 When school children are receiving care

2 When the client is pregnant, 5 When school children are receiving care The delegation process becomes more challenging when vulnerable populations such as pregnant women and school children are receiving care. Delegation can be safely and successfully carried out in the stable client. Delegation can be accomplished successfully when the geographical area is small and resources are abundant.

The healthcare team is caring for clients in an emergency department. How soon should the clients triaged as an emergency severity index-3 (ESI-3) be seen by the physician according to the five level triage system? 1 Immediately 2 Within 1 hour 3 Could be delayed 4 Within 10 minutes

2 Within 1 hour The clients triaged as an ESI-3 should be seen by the physician within an hour. The clients with life-threatening problems are triaged as ESI-1, and they should be provided with immediate care. The care of the clients in ESI-4 and ESI-5 can be delayed for some time because the condition of the clients in this level is stable. Clients with major injuries are triaged as an ESI-2 and require treatment within 10 minutes.

Which actions of the nurse exhibit transactional leadership? Select all that apply. 1 Motivating or inspiring the employees 2 Meeting the targets within the deadline 3 Working according to organizational rules 4 Correcting the errors in a reactive manner 5 Increasing the employee commitment of an organization

2, 3, 4 The characteristics of transactional leadership include valuing the orders and structures of an organization. The nurse who exhibits transactional leadership will meet the targets within the deadline given by the organization. The nurse will also follow the rules of an organization and will correct the errors of an employee in a reactive manner. Motivating or inspiring the employees and increasing employee commitment are the characteristics of transformational leadership.

For which situations would total client care be an appropriate delivery system? Select all that apply. 1 Client scheduled for lithotripsy for renal calculi 2 Client with an endotracheal tube for pulmonary sepsis 3 Client recovering from cardiovascular bypass graft surgery 4 Client recovering from the placement of a cerebrospinal fluid shunt 5 Client transferring to a rehabilitation unit after total hip replacement surgery

2, 3, 4 Total client care is used in critical care settings where one nurse provides total care to one or two critically ill clients. The client with an endotracheal tube for pulmonary sepsis and the client recovering from cardiovascular bypass graft surgery are considered acutely ill and will be receiving care in a critical care area. The client recovering from cranial surgery to place a cerebrospinal fluid shunt would be appropriate for total client care. Total client care would not be an ideal care approach for the client scheduled for lithotripsy or the client being transferred to a rehabilitation unit.

Which situations require that the registered nurse (RN) report to an appropriate authority? Select all that apply. 1. Client has a row of 3-inch circles down the back from "cupping" 2. Client is diagnosed with gonorrhea and requests not to report under the Health Insurance Portability and Accountability Act (HIPAA) 3. RN thinks a teenage client's signs are from abuse, but the health care provider does not 4. RN thinks an elderly client's signs are from abuse but the client denies this 5. Syphilis is diagnosed in an 11-year-old who denies sexual activity

2, 3, 4, 5 The RN is required to report suspected abuse of vulnerable clients (eg, underage, elderly, mentally ill) to appropriate authorities, regardless of what other practitioners think. A proper investigation, rather than conflicting opinions, will determine whether abuse has occurred (Option 3). The RN should report suspected abuse of vulnerable clients even if the client denies it because other factors (eg, dependence on the abuser, dementia) could be the reason for denial (Option 4). Sexually transmitted infection (STI) in a child is sexual abuse and must be reported and investigated (Option 5). The greater good of society outweighs an individual's right to confidentiality. Gonorrhea is an STI; the client should be informed that public health will be notified and partners will be contacted to receive treatment (Option 2). (Option 1) Cupping is a recognized alternative medicine practice in which a circular object is typically used to create suction underneath a cup. The tension pulls the skin upward and promotes release of muscle tension and scar tissue. After the process, the circular marks remain for a certain period. The location, organized rows, and history help validate the cause of the marks. Educational objective: An RN is required to report suspected abuse of vulnerable clients even if other practitioners do not agree or the clients deny it. An STI in a child is considered sexual abuse and requires reporting. Reportable conditions by law are not protected from reporting under the confidentiality of personal health care information in HIPAA.

The nurse has accepted a position as a navigator. What should the nurse expect to learn during training for this role? Select all that apply. 1 Approaches to reduce the cost of health care 2 Learning ways to work through the health systems 3 Strategies to teach clients about disease, including prevention and treatment 4 Assessing barriers that clients have encountered while attempting to receive care 5 Helping clients cope with delays in receiving treatment until barriers are overcome

2, 3, 4, 5 Navigators are responsible for overcoming health system barriers, providing health education about diseases from prevention to treatment, addressing barriers to care, and providing psychosocial support. These are the areas the nurse should expect to focus on when being trained in the role. Learning approaches to reduce the cost of health care would be appropriate for a case manager to learn.

Which skills are essential for the nurse who is setting priorities for client care? Select all that apply. 1 Evaluation 2 Assessment 3 Critical thinking 4 Case management 5 Clinical decision-making

2, 3, 5 Essential skills for the nurse who is setting priorities for client care include assessment, critical thinking, and clinical decision-making. Evaluation and case management are nursing skills; however, these are not essential for setting priorities.

The charge nurse is identifying tasks for staff caring for clients using functional nursing. Which tasks should be assigned to the licensed practical/licensed vocational nurse (LPN/LVN)? Select all that apply. 1 Showering two clients after breakfast 2 Administering topical and oral medications 3 Providing discharge instructions to three clients 4 Measuring capillary blood glucose level for five clients 5 Identifying nursing diagnoses after analyzing collected data

2, 4 Task assignments are made according to scope of practice. For the LPN/LVN, the scope of practice would include administering topical and oral medications and measuring capillary blood glucose levels. Showering would be a task assigned to unlicensed assistive personnel (UAP). Providing discharge instructions and identifying nursing diagnoses would be assigned to the registered nurse (RN).

Which aspects are considered when determining the willingness of a nursing assistant before delegating a task? Select all that apply. 1 Ability 2 Attitude 3 Personality 4 Confidence 5 Commitment

2, 4, 5 An individual's willingness is related to attitude, confidence, and commitment. Ability is related to knowledge and skills in a specific situation. Personality characteristics can influence the effectiveness of the delegator.

Which constructive aspects of conflict does the nurse leader identify? Select all that apply. 1 Polarizes groups 2 Releases pent-up emotions 3 Promotes ineffective communication 4 Helps individuals grow personally 5 Builds cohesiveness among people

2, 4, 5 Conflict serves as a release for pent-up emotion, anxiety, and stress. It helps individuals to grow personally and apply what they learn to future situations. It helps to build cohesiveness among people sharing the conflict, celebrating in its settlement and learning more about each other. It polarizes groups so they increase internal cohesiveness and reduce intergroup cooperation; this is a destructive aspect of conflict. Authentic communication is an aspect of conflict but not improbable communication.

The nurse delegates the tasks of caring for a postpartum client. During assessment, the nurse observes an infection in the client caused by lack of hygiene. Which member of the health care team is most likely responsible for the client's condition? Select all that apply. 1 Physician 2 Registered nurse 3 Licensed practical nurse 4 Licensed vocational nurse 5 Unlicensed assistive personnel

2, 5 The registered nurse may delegate the tasks of basic client care to the unlicensed assistive personnel (UAP), but is responsible and accountable for the outcome of the task. UAP are responsible for the client's basic hygiene and comfort. A physician may delegate the task of client care to the registered nurse and other assistive personnel. A licensed practical nurse (LPN) or licensed vocational nurse (LVN) generally carries out tasks such as administering oral medication, but is not typically responsible for client hygiene if UAP are available.

A nurse is providing anticipatory guidance to a client with early Alzheimer disease and osteoarthritis. Current symptoms include mild forgetfulness and cognition changes. Which is the best example of an educational goal for anticipatory guidance? 1. The client will demonstrate proper organization of medications in a weekly pill box by the end of the teaching session. 2. The client will identify and attend a support group meeting for clients with dementia by the end of the month. 3. The client will verbalize 2 home safety changes that can prevent falls during disease progression by the end of the session. 4. The client will verbalize 3 examples of easy, nutritious meals that can be prepared independently by the end of the clinic visit.

3 Anticipatory guidance prepares clients and caregivers for future health needs and is useful throughout life, from pediatric growth and development to anticipated changes related to disease processes. This type of education promotes health and helps to reduce client/caregiver stress and anxiety, which heighten with unexpected cognitive, physical, and emotional changes. Anticipatory guidance educational goals should be client-oriented, realistic, objective, measurable, and focused on preparing for future needs specific to the client. The client with Alzheimer disease and osteoarthritis is at high risk for falls with disease progression. In the early stage, the client can make changes in the home to promote safety in the future (Option 3). (Option 1) Memory aids (eg, pill organizers, alarms) should be used now, while the client has only mild cognition changes. As the disease progresses, a caregiver should take over medication management. (Option 2) Support groups are an appropriate intervention for current psychosocial needs (eg, depression). (Option 4) Clients with osteoarthritis are at risk for nutritional deficits due to functional decline (eg, inability to open jars), and clients with Alzheimer disease can forget to eat. The nurse should address this current need by teaching simple meal planning. Educational objective: Anticipatory guidance addresses expected changes related to growth and development or disease progression. Educational goals should be client-oriented, realistic, objective, measurable, and focused on preparing for future needs specific to the client.

What factors are most important for the nurse to consider when delegating responsibilities? 1 Preferences of the clients and staff 2 Physical layout of the unit and client rooms 3 Staff member's level of education and expertise 4 Client's diagnosis and length of time in the hospital

3 Delegation should provide for client safety based on staff capabilities as determined by level of education and experience. Although client and staff preferences may be considered, they are not the most important criteria for determining delegation of tasks. Although geographic factors may be considered when tasks are delegated, these are not the most significant criteria to consider. The client's acuity, not diagnosis or length of time in the hospital, is the most important client factor to consider when appropriate staff members are assigned to provide care.

The health care provider gives the preoperative nurse a signed consent form and walks away rapidly. The client turns to the nurse and states, "I don't know what is going on. Why do I need surgery?" What is the most appropriate action? 1. Call the nursing supervisor 2. Call the operating room scheduler and cancel the surgery 3. Page the health care provider and request clarification on behalf of the client 4. Report the incident to hospital administration

3 Informed consent requires that the health care provider performing the procedure explain everything to the client's satisfaction (within reason). Signed consent may be witnessed by the nurse. If the client does not fully understand informed consent, the nurse must notify the health care provider or refer up the chain of nursing command. The nurse is not responsible for verifying that the client understands the procedure and its respective risks. (Option 1) This would be appropriate if the health care provider refuses to talk to the client. (Option 2) This is not the nurse's responsibility; this request would have to be relayed up the chain of nursing command. (Option 4) This is premature; the incident is isolated and not all facts are known. Educational objective: Clients may not consent to an invasive procedure without being informed of the clinical reasoning, consequences, and possible complications.

The charge nurse must assign a room for a client who was transferred from a long-term care facility and is scheduled for extensive surgical debridement to remove infected tissue from an unstageable pressure injury. Which room assignment is the most appropriate for this client? 1. Room A: Client with multiple myeloma who is being treated with corticosteroids 2. Room B: Client with diabetes mellitus and osteomyelitis receiving IV antibiotics 3. Room C: Client with a gastrointestinal bleed who has a nasogastric tube 4. Room D: Client with influenza with a high fever who is receiving oseltamivir

3 Surgical debridement of an unstageable pressure injury involves using a scalpel to remove necrotic (eschar) or infected tissue from the wound to promote healing. The most appropriate room assignment for this client is Room C, as the client with a gastrointestinal bleed and nasogastric tube is the least susceptible to infection compared with the clients in Rooms A and B (Option 3). (Option 1) Multiple myeloma is a cancer that involves proliferation of malignant plasma cells (monoclonal antibodies), which are ineffective in providing protection against infection and suppress normal bone marrow cell production (eg, erythrocytes, platelets, leukocytes). This client in Room A is especially vulnerable to infection due to immunosuppression related to the disease process and to drug therapy with corticosteroids. (Option 2) The postoperative client should not be assigned to Room B with a client who has osteomyelitis, an infection of bone. (Option 4) The client with influenza requires droplet precautions and would likely require a private room (Room D). Clients with severe disease (ie, requiring hospitalization) should receive antiviral medication (eg, zanamivir, oseltamivir) as they are at high risk for complications. Educational objective: A client undergoing an extensive surgical debridement for an infected pressure injury should not be assigned to a room with a client who is vulnerable to infection (eg, immunocompromised) or who has an active infection.

The nurse notifies the health care provider of a change in client condition. Which of the following reports given by the nurse includes the most appropriate and complete information? 1. "A 43-year-old client with pneumonia in room 343 has wheezing, crackles, and diminished breath sounds. Temperature is 101.2 F (38.4 C), respirations are 36/min, and pulse oximeter shows 90%. I think the client may need arterial blood gas testing." 2. "A 75-year-old client in room 474 is in respiratory distress. The respiratory therapist (RT) gave a breathing treatment, but the client is deteriorating rapidly. The RT did not hear left side breath sounds and recommends a chest x-ray." 3. "An 80-year-old client in room 234 with a history of heart failure was admitted today for pneumonia and is receiving oxygen and antibiotics. The client is dyspneic and restless, and oxygen saturation is now 89%. Would you like me to increase the oxygen flow rate?" 4. "The client with pneumonia in room 265 is reporting shortness of breath. The RT gave the client a breathing treatment 30 minutes ago, but the client is no better. Would you like to prescribe any laboratory tests or make any changes to the treatment?"

3 The SBAR (Situation-Background-Assessment-Recommendation) provides a framework for communicating information about a change in client status to the health care provider (HCP). It includes the following information: S = Situation - what prompted the communication B = Background - pertinent information, relevant history, vital signs A = Assessment - the nurse's assessment of the situation R = Recommendation - request for prescription or action from the HCP The report given by the nurse in Option 3 contains the most appropriate and complete information. The nurse includes pertinent data related to history, admission, and present treatment (background); indicates when and what changes occurred (situation, assessment); and requests a prescription from the HCP (recommendation). (Option 1) This report does not include any information indicating a time frame for admission or when the change in condition occurred. (Option 2) This report does not include any information related to the admission time frame, current diagnosis, or pertinent data assessed by the nurse giving the report. (Option 4) This report does not include any information related to the admission time frame or pertinent data assessed by the nurse giving the report. Educational objective: Nurses commonly use the SBAR framework to report changes in client status to the health care provider, communicating the current situation, client background, nurse's assessment, and a recommendation for prescription or action.

An 84-year-old client with oxygen-dependent chronic obstructive pulmonary disease is admitted with an exacerbation and steady weight loss. The client has been in the hospital 4 times over the last several months and is "tired of being poked and prodded." Which topic would be most important for the nurse to discuss with this client's health care team? 1. Need for discharge to a skilled nursing facility 2. Nutritional consult with instructions on a high-calorie diet 3. Option of palliative care 4. Physical therapy prescription to promote activity

3 This client with advanced chronic obstructive pulmonary disease is approaching the end of life. The client has expressed the desire to avoid further tests, treatments, and hospitalizations. The goals of care should be consistent with the client's wishes and emphasize comfort and quality of life. Palliative care is appropriate for clients who wish to focus on quality of life and symptom management rather than life-prolonging treatments (Option 3). Palliative care may eventually include hospice care, after it is determined that the client has a life expectancy of less than 6 months. The nurse should advocate for the client and collaborate with members of the health care team to explore care options based on the client's wishes. (Option 1) This client has not clearly demonstrated a need for skilled nursing; additional assessment is needed to determine the most appropriate discharge setting. (Option 2) A high-calorie diet is appropriate for a client with weight loss, but many clients may have difficulty maintaining weight due to factors such as advanced disease and poor appetite. It is not the highest priority in this client, who is nearing the end of life and has expressed an interest in avoiding further testing and hospitalization. (Option 4) Physical therapy may be appropriate to help this client maintain current abilities. However, a client with disease this advanced is not likely to tolerate more activity or gain much additional functional capacity. Therefore, physical therapy is not the highest priority at this point. Educational objective: The client with an advanced, terminal disease (eg, chronic obstructive pulmonary disease) is often an appropriate candidate for palliative care. Palliative care emphasizes quality of life and symptom control and may eventually include hospice care based on the client's life expectancy.

The nurse manager is looking to include members of the health care team in the decision-making process but finds that many members are reluctant to offer ideas. Which approach by the manager generates a large volume of creative options while encouraging reticent members of the team to participate? 1 Applying the Delphi technique 2 Organizing focus groups 3 Planning brainstorming sessions 4 Applying nominal group techniques

3 Brainstorming helps to get a large volume of creative options from the team by providing a safe environment for the contribution of ideas. The application of the Delphi technique, focus groups strategy, and nominal group techniques are also effective in minimizing these problems, but do not provide a large volume of creative options. The Delphi technique involves collection and summarization of opinions and judgments on a particular issue from expert panels through interviews, surveys, or questionnaires. A focus group involves organizing moderated discussions with small groups of people to identify problems or evaluate processes; it is less likely to provide a nonjudgmental environment for generating ideas. A nominal group technique allows the opportunity to the team members to share ideas, but their ideas are judged and merits are given to each idea.

The registered nurse finds that two nursing students are arguing with each other. Which action by the registered nurse best represents a leadership quality? 1 Complaining to management and asking the students to go outside 2 Letting them continue arguing until they resolve the matter themselves 3 Assessing the condition and strategizing to resolve the matter by reducing the difference 4 Asking the reason for their argument and reprimanding them for having the argument in the hospital

3 Visioning is required by an efficient leader to engage others to assess the current reality. This skill includes determining and specifying a desired end-point state and then strategizing to reduce the differences. Letting them continue arguing and waiting until they resolve the matter may allow the argument to escalate. Complaining to management would pass off responsibility for a resolution and would not indicate leadership. Reprimanding the nurses may stop the argument, but would not be a constructive approach.

During a peer review, the manager finds that a newly appointed nurse leader encourages "bottom up" interaction among the workers. To which statement of the staff does the manager relate this? 1 "He/she accepts responsibility willingly." 2 "He/she is empathetic to the experiences of others." 3 "He/she involves us in shaping policies for client care." 4 "He/she believes in unleashing constructive energy rather than constraining energy."

3 "He/she involves us in shaping policies for client care." "Bottom-up" interaction values every nurse as a human resource with rich perspective. Therefore involving peers in shaping policies is an example of a "bottom-up" interaction. Willingness to accept responsibility is a generally desired attribute. Empathy to others' experiences shows emotional intelligence. A leader who unleashes constructive energy demonstrates the concept of focus on emergence.

Which statement would most likely be made by a client who is receiving care through the functional method? 1 "My nurse's name is Amy." 2 "Susan will be caring for me until 7 pm." 3 "I have to wait until the medication nurse is available for a pain pill." 4 "Barbara coordinated all of my care but I might receive the care from other nurses."

3 "I have to wait until the medication nurse is available for a pain pill." In the functional method, staff are assigned tasks according to scope of practice. In this approach a medication nurse will be identified. Knowing the nurse's name would more likely be a characteristic of team or primary nursing. Knowing which nurse will provide care for a specific period of time would indicate the case approach to care. Knowing which nurse coordinated the care that might be provided by others would more likely indicate primary nursing or case management.

The registered nurse is teaching a newly hired nurse about communicating with delegatees during delegation. Which statement made by the newly hired nurse indicates the need for further learning? 1 "I should identify priorities." 2 "I should specify deviations." 3 "I should provide examples of each delegation." 4 "I should specify any performance limitations to the delegatee."

3 "I should provide examples of each delegation." The delegator should ask the delegatee to provide examples of each delegation in order to understand his or her knowledge level. The delegator should identify priorities before delegating the task to the delegatee. The delegator should specify deviations, such as when the delegatee must take an immediate action while performing the task. The delegator should specify the performance limitations to the delegatee, which can help in performing the task effectively.

A nurse leader is teaching about the functional model of nursing to a student nurse. Which statements made by the student nurse indicate effective learning? Select all that apply. 1 "I will focus on one nurse caring for one client." 2 "I will accentuate the use of team collaborations." 3 "I will focus on tasks and activities allotted to me." 4 "I will provide comprehensive and coordinated nursing service." 5 "I will coordinate the unlicensed personnel to provide care to a large group of clients."

3 "I will focus on tasks and activities allotted to me." 5 "I will coordinate the unlicensed personnel to provide care to a large group of clients." In the functional model of nursing, one mainly focuses on performing tasks and activities that are allotted to him or her. Coordinating with unlicensed personnel to provide care to a large group of clients indicates the functional model of nursing. Focusing on one nurse caring for one client is seen in the case method of client care delivery. Concentrating on the use of team collaborations indicates the model analysis of primary nursing of client care delivery. Providing comprehensive and coordinated nursing services indicates the primary nursing model.

A client's chest tube has accidentally dislodged. What is the nursing action of highest priority? 1 Place the client in a left side-lying position. 2 Apply oxygen via nonrebreather mask. 3 Apply a petroleum gauze dressing over the site. 4 Prepare to reinsert a new chest tube.

3 Apply a petroleum gauze dressing over the site. A petroleum gauze dressing will prevent air from being sucked into the pleural space, causing a pneumothorax. The petroleum gauze dressing should be taped only on three sides to allow for excessive air to escape, preventing a tension pneumothorax. The physician should immediately be notified and the client assessed for signs of respiratory distress. Positioning the client on the left side will not make a difference in outcome. There is no indication that the client is experiencing respiratory distress. Preparing to reinsert a new chest tube is not a priority of the nurse at this moment.

Which nursing action allows for a thorough assessment of a trauma client to prioritize the client's care? 1 Avoiding manipulation of the client's limbs 2 Asking a family member about any client drug allergies 3 Cutting fabric that is stuck to the client's skin with scissors 4 Auscultating heart and lung sounds through the client's clothing

3 Cutting fabric that is stuck to the client's skin with scissors The nurse should remove all clothing to allow for a thorough assessment of the trauma client in order to accurately prioritize care. Cutting fabric that is stuck to the client's skin with scissors is the appropriate action by the nurse. It is necessary to avoid manipulation of the client's limbs during the trauma assessment. While it is important to ask a family member about any client drug allergies, this is done after the initial assessment of the client. Clothing is always removed to allow for an accurate trauma assessment.

A registered nurse (RN) delegates the task of foot care for a client to an unlicensed nursing personnel (UNP). The UNP is skillful and willing to perform the given task, but was recently hired and is unfamiliar with the client's condition. What should the RN do in this situation? 1 Provide guidance to the UNP. 2 Observe and motivate the UNP. 3 Establish mutual expectations and conditions. 4 Explain what to do and how to perform the task.

3 Establish mutual expectations and conditions. If the delegatee is new to the workplace, but has the ability and willingness to perform a task, the RN should establish mutual expectations and conditions of performance to establish a good relationship. If the delegatee has limited knowledge and ability to perform a task, the delegator is expected to guide the UNP. The RN is expected to observe and monitor the task performed by the delegate to ensure the delegatee has the ability and willingness to establish a relationship and accomplish the work. The RN can also explain the task and how to execute it if the situation involves a new task and relationship is ongoing.

The unit nurse manager comes to work obviously intoxicated. Which action is the staff nurse ethically obligated to take? 1 Call the security guard. 2 Tell the nurse manager to go home. 3 Have the supervisor validate the observation. 4 Offer the nurse manager a large cup of coffee

3 Have the supervisor validate the observation. The staff nurse should call the hospital supervisor to confirm and handle the problem. The security guard has no authority in this situation. Although sending the nurse manager home removes the nurse manager from the clinical setting, this action misses an opportunity for documenting the situation and the nurse manager is in no condition to drive safely. Drinking coffee does not make a person less intoxicated.

The registered nurse (RN) is caring for a client with renal calculi. Which healthcare professional is most suitable to be delegated the task of administering urinary alkalinizer by mouth to the client? 1 Certified technician 2 Patient care associate 3 Licensed practical nurse 4 Unlicensed assistive personnel

3 Licensed practical nurse Administering oral medications such as urinary alkalinizer can be safely delegated to a licensed practical nurse (LPN) or licensed vocational nurse (LVN) as per guidelines. Certified technician is a licensed assistive personnel whose scope of practice is limited for administering medications. The scope of practice of the patient care associate and unlicensed assistive personnel is limited to performing basic care, feeding, and hygiene.

Which intervention should the charge nurse perform to manage a task when the unlicensed nursing personal (UNP) is lacking competence? 1 Provide constructive feedback and assign another task. 2 Provide open vocal feedback and withdraw the UNP from task. 3 Provide additional support and temporarily lower expectations. 4 Provide accountability to the organization and undergo training again.

3 Provide additional support and temporarily lower expectations. The charge nurse should manage a task by temporarily lowering expectations and providing additional support to an unlicensed nursing personal (UNP) who is lacking competence. This strategy allows the UNP to build on strengths, minimize weaknesses, and gain confidence. Providing constructive feedback and assigning another task may not improve the strategy of the UNP and may affect the productivity of the new task. Withdrawing the UNP from the task and providing open vocal feedback may weaken the working relationship and the UNP may lose confidence or become frustrated. The charge nurse should be aware of accountability to the organization in case of a failure of the delegation, but assigning the UNP for further training may lead to high risk for legal liability.

The registered nurse assigns a task to a newly hired licensed practical nurse (LPN) to monitor a client's blood pressure. Which is relevant to the delegator in this situation? 1 Requires explanation 2 Requires little guidance 3 Requires more guidance 4 Requires creating mutual expectations

3 Requires more guidance When the task is assigned to a newly hired LPN, the registered nurse should provide more guidance to the LPN. The LPN requires explanation when there is an ongoing process and the task is already assigned. The LPN requires little guidance when the LPN has expertise in work and performs in a correct way. The delegator and the delegatee should have mutual expectations when there is ability and willingness but the relationship is new.

Which element creates an integrative process that fosters effective delegation decisions by the registered nurse? 1 Ability 2 Liability 3 Stability 4 Ethnicity

3 Stability Stability is the element that creates an integrative process that fosters effective delegation decisions. Ability is the factor that needs to be assessed to determine the level the leaders determine. Liability is the person's responsibility and accountability for individual actions. Ethnicity does not play a role in the process of delegation.

Which action of the nurse leader indicates implementing Gardner's task of "explaining"? 1 Assisting clients and families in formulating their vision of future well-being 2 Providing self-care to enhance the ability to care for staff, clients, and their families 3 Teaching and interpreting the information to ensure clients' functioning and well-being 4 Assisting clients in sorting out and articulating personal values in relation to health problems

3 Teaching and interpreting the information to ensure clients' functioning and well-being The nurse leader implements Gardner's task of "explaining" by teaching and interpreting the information that ensures clients' functioning and well-being. The nurse leader implements Gardner's task of "envisioning goals" by assisting clients and their families in formulating their vision of future well-being. The nurse leader implements Gardner's task of "renewing" by providing self-care to enhance the ability to care for staff, clients, and their families. The nurse leader implements Gardner's task of "affirming values" by assisting clients and their families as they sort out and articulate personal values in relation to health problems.

Which critical factors would the nurse say are associated with situational-contingency theory? Select all that apply. 1 Environmental demands 2 The personal characteristics of followers 3 The ability of the leader to reward followers 4 The degree of trust and respect between leaders and followers 5 The task structure about goals and the complexity of the problems being faced

3 The ability of the leader to reward followers 4 The degree of trust and respect between leaders and followers 5 The task structure about goals and the complexity of the problems being faced Situational-contingency theory considers three factors to be critical. It includes the ability of the leader to reward followers in terms of power and position. It is also comprised of the degree of trust and respect between leaders and followers. The task structure about goals and the complexity of problems faced is a critical factor. Environmental demands and the personal characteristics of followers are contingent variables.

Why does a nurse manager assign a resource person in a healthcare organization? 1 To delegate tasks 2 To supervise actions 3 To serve as a mentor 4 To reassign duties to workers

3 To serve as a mentor The nurse manager assigns a resource person in a healthcare organization to serve as a mentor for the agency nurse to prevent potential problems that could arise if the staff member does not know the institutional routine. The nurse manager also delegates a task, supervises actions, and reassigns duties to workers.

The charge nurse is assigning client care to oncoming staff. The new nurse on the unit expresses an inability to care for the assigned client and is requesting to be reassigned. What should the delegator do in this situation? 1 Provide little guidance to the delegatee 2 Evaluate the ability and willingness of the delegate. 3 Understand the delegatee's motivation in the situation. 4 Understand the kind of support needed to accomplish the task.

3 Understand the delegatee's motivation in the situation. An inability to care for the assigned client shows inability and/or unwillingness in the new nurse. The delegator needs to understand the delegatee's motivation related to the situation and should use selling leadership style, which leads to a supportive relationship. Little guidance should be provided to the delegatee when he or she has high ability and willingness to perform the task delegated efficiently. When a new team begins to work together, the delegator should evaluate the ability and willingness of the delegatee. In the application of practice before delegation, the delegator needs to understand the kind of support to be given to accomplish the task.

Which factors does the registered nurse consider in the decision to delegate process? Select all that apply. 1 Evaluation 2 Nursing judgment 3 Predictability of outcomes 4 Pervasive functions of assessment 5 Complexity of the task to be performed

3, 5 Two factors responsible for the decision to delegate process are predictability of outcomes and complexity of the task to be performed. Evaluation, nursing judgment, and pervasive functions are not factors in the decision to delegate process.

The nurse enters a client's room just as the unlicensed assistive personnel (UAP) is completing a bath and placing thigh-high anti-embolism stockings on the client. Which situation would cause the nurse to intervene? 1. UAP applies the anti-embolism stockings while maintaining the client in supine position 2. UAP carefully smoothes out any wrinkles over the length of the stockings 3. UAP checks that the toe opening of the stockings is located on the plantar side of the foot 4. UAP rolls down and folds over the excess material at the top of the stockings

4 Anti-embolism stockings are part of venous thromboembolism (VTE) prophylaxis in hospitalized clients. Anti-embolism stockings improve blood circulation in the leg veins by applying graduated compression. When fitted properly and worn consistently, the stockings decrease VTE risk. The stockings should not be rolled down, folded down, cut, or altered in any way. If stockings are not fitted and worn correctly, venous return can actually be impeded. (Option 1) Anti-embolism stockings should be applied before ambulating while the client is in bed; this maximizes the compression effects of the stockings and promotes venous return. The UAP has performed this correctly. (Option 2) Wrinkles should be smoothed out to avoid impeding venous return. The UAP has performed this correctly. (Option 3) The toe opening should be located on the plantar side of the foot/under the toes. The UAP has performed this correctly. Educational objective: Anti-embolism stockings are worn by clients as part of VTE prophylaxis. It is important that the nurse verifies the stockings are correctly fitted and worn appropriately. Incorrect size and fit or alterations to the stockings can impede venous return.

A major disaster involving hundreds of victims has occurred, and an emergency nurse is sent to assist with field triage. Which client should the nurse prioritize for transport to the hospital? 1. Client at 8 weeks gestation with spotting and pulse of 90/min 2. Client with a compound femoral fracture and an oozing laceration 3. Client with fixed and dilated pupils and no spontaneous respirations 4. Client with paradoxical chest movement throughout respirations

4 Disaster triage is based on the principle of providing the greatest good for the greatest number of people. Clients are triaged rapidly using a color-coded system to categorize them from highest medical priority (emergent) to lowest (expectant). The client with flail chest (ie, paradoxical chest movement during respiration) from multiple fractured ribs is at risk for respiratory failure from impaired ventilation. In addition, mobile fractured ribs may puncture the pleura or vessels, causing hemothorax and/or pneumothorax at any time. Therefore, this client would be classified as emergent due to airway compromise, which requires immediate treatment (Option 4). (Option 1) Spotting at 8 weeks gestation may indicate complications of pregnancy (eg, miscarriage, ectopic pregnancy, hydatidiform mole). With stable vital signs, this client would be classified as nonurgent as the fetus is not at the age of viability and there is no evidence of risk to the mother's life. (Option 2) The client with a compound fracture and oozing laceration would be classified as urgent and require care within 2 hours to prevent life-threatening complications (eg, hemorrhagic shock). (Option 3) Absent respirations and fixed pupils indicate severe neurologic damage or death. Therefore, this client would be classified as expectant. Educational objective: During mass casualty events, the goal is the greatest good for the greatest number of people. Clients are triaged rapidly using a color-coded system that categorizes them from highest medical priority to lowest: red (emergent), yellow (urgent), green (nonurgent), and black (expectant).

Interdisciplinary client care rounds and hand-off communication are examples of strategies used to improve communication in health care settings. What is the most important outcome of effective communication among care givers? 1. Decreased length of hospital stay 2. Less obvious needs of clients met accordingly 3. Properly educated clients 4. Reduced number of medical errors

4 Miscommunication between health care providers may cause serious medical errors when clients are handed off or transferred. Medical errors can be effectively reduced by employing strategies (eg, Situation, Background, Assessment, and Recommendation [SBAR] reporting technique, nurse-to-nurse change of shift reports, multi-professional bedside rounds) to improve communication and collaboration. Nurses should be as proficient in their communication skills as they are in their clinical skills. (Options 1, 2, and 3) Improved communication may aid in assessing a client's educational needs and meeting less obvious needs; it can also contribute to a shorter length of stay. However, these are not the most important outcomes. Educational objective: Effective communication among caregivers is necessary to deliver safe client care and reduce the number of medical errors.

A charge nurse suspects that the unlicensed assistive personnel (UAP) is falsifying the documentation of clients' capillary glucose results rather than performing the test. What is the best action by the charge nurse to handle this situation? 1. Ask a client if the UAP has performed the test 2. Discuss the importance of task completion and accurate documentation in a staff meeting 3. Give the UAP a verbal warning not to falsify data 4. Take a client's capillary glucose personally and compare it to the recorded result

4 The best initial result is to assess and validate the charge nurse's perception. Doing the test and comparing results randomly/intermittently will give data to prove/disprove this concern. (Option 1) It could cause concern to involve a client when there may be an issue about inadequate provider care. The nurse should handle it independently. (Option 2) It is good to reinforce policies in general announcements to the entire staff, especially if wide-spread compliance is a concern. However, there is only one person that is suspected of not adhering in this case. Speaking out is often a general step taken, but the intended individuals usually don't hear the information. In addition, this is information that the staff has known/heard before. (Option 3) The normal discipline process is a verbal warning, a written warning, suspension, and termination. To initiate the process, there has to be evidence of wrong doing. However, it is only a suspicion at this point. Educational objective: When deliberate inaccurate documentation is suspected, gather evidence before confronting the staff member. One way of doing this is by checking the data personally and comparing it to what has been documented.

The charge nurse on the medical surgical unit must assign a room for an immediate post-operative nephrectomy client. Which room assignment is the best option for this client? 1. Room 1 - Client with diabetes mellitus and chronic kidney disease who is on hemodialysis and has a serum glucose level of 265 mg/dL (14.7 mmol/L) 2. Room 2 - Client with chronic HIV infection and overwhelming fatigue who has a CD4+ cell count of 200/mm3 (0.2 x 109/L) 3. Room 3 - Client with cellulitis of the leg due to a spider bite who has a white blood cell count of 13,000/mm3 (13.0 x 109/L) 4. Room 4 - Client with severe epistaxis due to a traumatic nasal fracture who has a platelet count of 85,000/mm3 (85 x 109/L)

4 The best option is room 4 with the client who has severe epistaxis and decreased platelet count (normal 150,000-400,000/mm3 [150-400 x 109/L]) as this does not place the immediate post-operative client at increased risk for infection. (Options 1, 2, and 3) The clients in these rooms place the postoperative client at increased risk for infection: - Room 1: A client with diabetes mellitus and advanced chronic kidney disease may have infectious complications due to increased susceptibility to infection resulting from an altered immune response and decreased leukocyte function due to hyperglycemia. In addition, hemodialysis increases the risk for infection due to invasive lines and catheters. - Room 2: A low CD4+ cell count (<500/mm3 [0.5 x 109/L], normal is 500-1,200/mm3 [0.5-1.2 x 109/L]) in a client with chronic HIV infection indicates disease progression. It can also indicate progression of asymptomatic early infections to more advanced symptomatic infections. - Room 3: The client with cellulitis and an increased white blood cell count (>11,000/mm3 [11.0 x 109/L]) has an infection. Educational objective: An immediate post-operative client should not be assigned a bed in a room with a client who is contagious or potentially infected as this poses an increased risk for infection.

Which performance appraisal method is used by nurse managers to measure the performance of the nurse both qualitatively and quantitatively? 1 Narrative method 2 Critical incidents method 3 Graphic rating scale form 4 Behaviorally anchored rating scales

4 The behaviorally anchored rating scale is a performance appraisal method that provides an overview of nursing standards by determining performance both qualitatively and quantitatively. The narrative method requires a manager to write a statement about the nurse's performance. The critical incidents type is a performance appraisal method in which a manager keeps a written record of positive and negative performance. The graphic rating scale form is a performance appraisal checklist on which a manager rates the nurse's performance on a continuum such as excellent, good, average, and poor.

The registered nurse is caring for a client admitted to the hospital with chronic obstructive pulmonary disease. Which assessment by the registered nurse before delegating would help to determine the principle of "right person"? 1 "Is the environment conducive for completing the task safely?" 2 "Does the licensed practical nurse (LPN) know about polices of the institution?" 3 "Can the nursing assistive personnel (NAP) evaluate the client's' condition appropriately?" 4 "Does the nursing assistive personnel (NAP) have the knowledge and expertise to perform the task?"

4 "Does the nursing assistive personnel (NAP) have the knowledge and expertise to perform the task?" In this situation, "Does the NAP have knowledge and expertise to perform the task?" is related to the right person for delegation. The right person deals with expertise and experience, knowledge and skills. The assessment, "Is the environment conducive for completing the task safely?" is related to right circumstance. The assessment, "Does the LPN know about polices of the institution?" is related to right task. The assessment, "Can the NAP evaluate the client's condition appropriately?"is related to right supervision.

The registered nurse is evaluating the statements made by a student nurse after teaching about the delegation communication template. Which statement made by the student nurse indicates a need for correction? 1 "The delegation communication template should identify the priorities." 2 "The delegation communication template should specify the deviations." 3 "The delegation communication template should identify the appropriate resources." 4 "The delegation communication template should include only the work that is to be delegated."

4 "The delegation communication template should include only the work that is to be delegated." The delegation communication template should emphasize the work that is not to be delegated. T he delegation communication template should identify the priorities such as what is to be done first. The delegation communication template should specify deviations such as when to take an immediate action. The delegation communication template should identify the appropriate resources including the individual consultants.

Under a leader, a team of followers has failed to achieve success in conducting research. What does an effective leader do in this situation? 1 Criticizes the team members for failure 2 Provides excuses for the negative outcome 3 Refuses to take the responsibility for failure 4 Accepts failure and gains experience from it

4 Accepts failure and gains experience from it An effective leader should accept the failure and gain experience from it to avoid repetition of the same errors. The effective leader should counsel and motivate the team members for future success and should not criticize them. Making excuses for the negative outcome is not the correct action of the effective leader. The effective leader should take responsibility for the failure and the growth he or she will gain from it.

The nurse as a leader assists multidisciplinary leaders to achieve optimal functioning to benefit client care delivery. Which Gardner's task is the nurse applying? 1 Managing 2 Developing trust 3 Serving as a symbol 4 Achieving workable unity

4 Achieving workable unity When the nurse assists multidisciplinary leaders to achieve optimal functioning to benefit client care delivery, it indicates the Gardner's task of achieving workable unity. Relating to and inspiring staff, management, and community leaders to achieve desired levels of health and well-being indicates Gardner's task of managing. When the nurse is honest in role performance, it indicates Gardner's task of developing trust. When the nurse represents the values and beliefs of the organization to internal and external constituents, it indicates Gardner's task of serving as a symbol.

Which behavior of the nurse indicates management skills according to Gardner's task of achieving workable unity? 1 Assisting the client/family with planning, priority setting, and decision making 2 Assisting multidisciplinary leaders to achieve optimal functioning to benefit client care delivery 3 Assisting clients to achieve optimal functioning to benefit the transition to enhanced health functions 4 Assisting staff to achieve optimal functioning to benefit the transition to enhanced organizational functions

4 Assisting staff to achieve optimal functioning to benefit the transition to enhanced organizational functions According to Gardner's task of achieving workable unity, the nurse in a management position helps the staff achieve optimal functioning to benefit transition to enhanced organizational functions. Assisting the client/family with planning, priority setting, and decision making is the clinical position of Gardner's task of managing. Assisting multidisciplinary leaders to achieve optimal functioning to benefit client care delivery is the executive position of Gardner's task of achieving workable unity. Assisting clients to achieve optimal functioning to benefit the transition to enhanced health functions is the clinical position of Gardner's task of achieving workable unity.

The nurse is assisting a client out of bed. Which is the priority nursing action? 1 Monitoring the client's blood pressure 2 Assessing the client's level of consciousness 3 Ensuring the call bell is within the client's reach 4 Assisting the client from a supine to an upright position

4 Assisting the client from a supine to an upright position The priority action to ensure the client's safety when assisting a client during a transfer is to assist the client from a supine to an upright position. Monitoring blood pressure and assessing the level of consciousness nursing actions that should be completed prior to assisting the client out of bed. Ensuring the call bell is within the client's reach is a nursing action that should be completed each time the nurse leaves the client's room.

Which client's care is least likely to be delegated to unlicensed nursing personnel (UNP)? 1 Client A - paraplegia, home care 2 Client B - femur fracture in cast, extended care 3 Client C - Alzheimer's, long-term care 4 Client D - accidental poisoning, emergency care

4 Client D - accidental poisoning, emergency care UNP can safely care for clients in stable condition because these cases will not require critical nursing assessment and decision-making. In an emergency setting, the client with accidental poisoning should be kept under constant monitoring and any small detail of fluctuation should be noted. Therefore, client D cannot be safely delegated to UNP. Clients undergoing home care, extended care, or long-term care do not require exhaustive monitoring and do not have life-threatening conditions, so they can be safely delegated to UNP.

A case manager telephones a client with lung disease to evaluate respiratory function with home oxygen use and then contacts the primary care provider. What is the purpose of this communication? 1 Collect data 2 Plan activities 3 Validate interventions 4 Evaluate expected outcomes

4 Evaluate expected outcomes It is essential that the case manager have frequent interaction with the client and the healthcare provider to achieve and evaluate expected outcomes. The nurse is not phoning the client and care provider to collect data, plan activities, or validate interventions.

The emergency department (ED) nurse is providing care to a burn trauma client. Which is the priority for the nurse to monitor for after removing the client's clothing? 1 Bradypnea 2 Bradycardia 3 Hypotension 4 Hypothermia

4 Hypothermia After the removal of the burn client's clothing, the priority for the nurse is to monitor for hypothermia because burn trauma clients lose their ability to maintain body temperature due to the loss of skin which acts as an insulator. While the nurse will monitor for bradypnea, bradycardia, and hypotension, hypothermia is the priority.

The nurse is teaching breathing exercises to a client who underwent surgery. Which member of the healthcare team is most suitable for reinforcement of teaching in the client? 1 Certified technician 2 Case manager 3 Cross-trained technician 4 Licensed vocational nurse (LVN)

4 Licensed vocational nurse (LVN) The licensed practical nurses (LPN) are most suitable to be delegated the task of reinforcement teaching. A certified technician is an unlicensed member who can only record the vital signs or provide basic hygiene to the client. A case manager can provide primary education to the client. These personnel can delegate the task of reinforcement teaching to the assistive nursing personnel. Cross-trained technicians may perform respiratory therapy, draw blood samples, and monitor electrocardiography.

A client with myocardial infarction is admitted in the emergency department, and the primary health care provider recommended the placement of a stent. The client is incompetent to understand the situation. What model does the nurse manager think would be beneficial in this situation? 1 Decision model 2 Autonomy model 3 Social justice model 4 Patient-benefit model

4 Patient-benefit model The patient-benefit model uses substituted judgment such as determining what the client would want for himself or herself if capable of making these issues known, and thereby facilitates decision making for incompetent clients. The decision model is used for nurses; it depends on specific circumstances to know if the situation is routine and predictable or complex and uncertain. The autonomy model facilitates decision making for competent clients. The social justice model considers broad social issues and is accountable to the overall institution.

While delegating a task, the registered nurse says to the delegatee, "Please tell me how you go about performing this procedure, and I will share with you my perspective about how frequently and under what conditions we need to communicate with and report to each other." What does this indicate? 1 The delegatee requires explanation. 2 The delegatee has limited knowledge. 3 The delegatee and delegator relationship is established. 4 The delegatee and delegator are creating mutual expectations.

4 The delegatee and delegator are creating mutual expectations. Asking a delegatee how he or she performs the procedure and sharing each other's perspective and when to communicate indicates that the delegator and delegatee are creating mutual expectations. When the delegatee demonstrates a task, it indicates that the delegatee requires explanation. The delegator will direct the procedures to the delegatee when the delegatee has limited knowledge. The delegator will motivate the delegatee when the relationship between the delegatee and delegator is established.

A healthcare team is caring for a client with diabetes insipidus. According to the functional model, which healthcare personnel would the nurse state is qualified to perform all hygienic tasks? 1 Registered nurse 2 Licensed practical nurse 3 Licensed vocational nurse 4 Unlicensed assistive personnel

4 Unlicensed assistive personnel Unlicensed assistive personnel perform all the hygiene tasks. Per the functional model, intravenous (IV) medication administration is provided by the registered nurse. The licensed practical nurse and a licensed vocational nurse may be permitted to give IV administration of medications, but they maybe limited to giving oral medications.

Which healthcare professionals would the nurse say work under active delegation? Select all that apply. 1 Physician examining clients in the outpatient ward 2 Nurse giving medication to a client as prescribed by the physician 3 Pharmacist dispensing medicines prescribed by the physician to the client 4 Licensed practical nurse (LPN) administering oral medications as delegated by the registered nurse (RN) 5 Unlicensed nursing practitioner (UNP) maintaining oral hygiene of the clients as delegated by the registered nurse

4, 5 If tasks are delegated to a delegatee after a thorough assessment by the delegator who remains accountable for the delegated tasks, it is called active delegation. Delegation to LPNs, UNPs, and other nursing personnel by the registered nurse are examples of active delegation, such as administering oral medications and maintaining oral hygiene. Any task performed normally, or as an essential part of practice by licensed individuals such as physicians, nurses, and pharmacists form a part of passive delegation.


Set pelajaran terkait

Server+ Chapter 5 Review Questions

View Set

Domain 3 Data analytics and informatics

View Set

Business 230- Exam #2 (Questions)

View Set

Midpoint and Distance in the Coordinate Plane Practice

View Set